LSAT and Law School Admissions Forum

Get expert LSAT preparation and law school admissions advice from PowerScore Test Preparation.

User avatar
 Adam354
  • Posts: 29
  • Joined: Feb 08, 2022
|
#93772
I do not understand the difference between

L closes M must be in
&
L closes R must be in

L out -> N out -> R in -> M in

Therefore, both L out R in, and L out M in seem to have the same effect.

So A & B both seem correct. What am I missing?
 Adam Tyson
PowerScore Staff
  • PowerScore Staff
  • Posts: 5390
  • Joined: Apr 14, 2011
|
#93813
What you're missing there, Adam, is that the question told us to replace the rule about R and M. If that rule is removed from the game, M becomes a random variable with no rules affecting it. Answer choice B does nothing to change that, so M remains random, and in your scenario there is no reason that M must be in. Whatever else the correct does, it must have some direct impact on M!
User avatar
 SGD2021
  • Posts: 72
  • Joined: Nov 01, 2021
|
#94609
Hello, I am confused between A and D.

A) says that Lc-->Mo. We know that if L is closed, then N is closed and if N is closed then R is open. Why does this mean Ro--> Mo?

D says Lo-->Mc. We know that if L is open then R must be closed. So why can't this be read as Mc-->Rc (which would be the contrapositive of the rule we're replacing?) . Why do we read it as Rc-->Mc?
 Robert Carroll
PowerScore Staff
  • PowerScore Staff
  • Posts: 1819
  • Joined: Dec 06, 2013
|
#94687
SGD2021,

To your first point - use the last rule:

If R is open, L is closed (last rule)

If L is closed, M is open (answer choice (A))

By a chain of conditionals, if R is open, M is open, which is what we wanted.

To your second point:

Answer choice (D) isn't even true, much less a good substitute for the third rule. Look at question #15, answer choice (C) - this is a wrong answer for question #15, and says "M stays open". In other words, we already know from this answer that L and M could be open at the same time - so what answer choice (D) is saying for question #17 isn't true at all.

It looks like you're doing the Mistaken Reversal of "Lo-->Mc". You say that we know that if L is open, then M is closed. And if L is open, R is closed. So if L is open, two different necessary conditions happen - but they're both necessary conditions. They don't connect to each other at all.

Robert Carroll

Get the most out of your LSAT Prep Plus subscription.

Analyze and track your performance with our Testing and Analytics Package.